Difference between revisions of "2022 AMC 12B Problems/Problem 3"
(Added third, nonrigorous solution) |
m (→Solution 3 (Not Rigorous): - signed name) |
||
Line 22: | Line 22: | ||
== Solution 3 (Not Rigorous) == | == Solution 3 (Not Rigorous) == | ||
Because <math>121 = 11*11</math>, because <math>11211</math> has sum of digits <math>6</math> (and therefore is divisible by <math>3</math>), and because <math>1112111 = 1100011 + 121</math> (a multiple of <math>11</math>), none of the first three numbers in the sequence are prime. Therefore, if the answer were anything other than <math>\boxed{\textbf{(A) } 0}</math>, determining the answer conclusively would require proving that some positive integer <math>111121111</math> or greater is prime, an extremely time-consuming task given the conditions of the test. | Because <math>121 = 11*11</math>, because <math>11211</math> has sum of digits <math>6</math> (and therefore is divisible by <math>3</math>), and because <math>1112111 = 1100011 + 121</math> (a multiple of <math>11</math>), none of the first three numbers in the sequence are prime. Therefore, if the answer were anything other than <math>\boxed{\textbf{(A) } 0}</math>, determining the answer conclusively would require proving that some positive integer <math>111121111</math> or greater is prime, an extremely time-consuming task given the conditions of the test. | ||
+ | |||
+ | ~Orange_Quail_9 | ||
== See Also == | == See Also == | ||
{{AMC12 box|year=2022|ab=B|num-b=2|num-a=4}} | {{AMC12 box|year=2022|ab=B|num-b=2|num-a=4}} | ||
{{MAA Notice}} | {{MAA Notice}} |
Revision as of 11:57, 21 November 2022
Problem
How many of the first ten numbers of the sequence , , , ... are prime numbers?
Solution 1
Write , , . It becomes clear that of these numbers are prime.
In general, (where there are 's on either side of the ) can be written as , where the first term has 's.
Solution 2
Let denote the digit written times and let denote the concatenation of , , ..., .
Observe that
Since for all positive integers and , is equal to
Both terms are integers larger than since , so of the numbers of the sequence are prime.
Solution 3 (Not Rigorous)
Because , because has sum of digits (and therefore is divisible by ), and because (a multiple of ), none of the first three numbers in the sequence are prime. Therefore, if the answer were anything other than , determining the answer conclusively would require proving that some positive integer or greater is prime, an extremely time-consuming task given the conditions of the test.
~Orange_Quail_9
See Also
2022 AMC 12B (Problems • Answer Key • Resources) | |
Preceded by Problem 2 |
Followed by Problem 4 |
1 • 2 • 3 • 4 • 5 • 6 • 7 • 8 • 9 • 10 • 11 • 12 • 13 • 14 • 15 • 16 • 17 • 18 • 19 • 20 • 21 • 22 • 23 • 24 • 25 | |
All AMC 12 Problems and Solutions |
The problems on this page are copyrighted by the Mathematical Association of America's American Mathematics Competitions.